When thinking about an unrelated problem, I wanted to sum
$$\sum_{k\in[\sqrt n,2\sqrt n]}\#\{j\leq k/5:\gcd(k,j)=1\}$$and I wanted this sum to be$\Theta(n)$. From the wikipediaI can see that if we take away the restriction of$\leq n/5$ we do have our result. A natural definition to consider given this is the following
$$\varphi(n,\varepsilon):=\#\{k\leq\varepsilon n:\gcd(k,n)=1\}$$
My question is the following
Question. Can we prove that for every fixed$\varepsilon\in(0,1)$ we have$\varphi(n,\varepsilon)=\Theta(\varphi(n))$?
I've been trying to do an averaging argument to prove that it holds true for almost every$n$ (which at least would've been enough for my original application), but I'm not too well-versed in number theory so I haven't been succesful
- $\begingroup$I don't know (yet?) about $\varphi(n,\varepsilon)$, but I can prove that your sum is $\Theta(n)$ for arbitrary values of $\frac{1}{5}$. Would you consider that an answer?$\endgroup$Dermot Craddock– Dermot Craddock2025-11-20 20:26:54 +00:00CommentedNov 20 at 20:26
- $\begingroup$@DermotCraddock sure!$\endgroup$Bruno Andrades– Bruno Andrades2025-11-20 20:37:39 +00:00CommentedNov 20 at 20:37
1 Answer1
The answer is yes. Almost surely there's an answer already on this site, but since I couldn't find it after searching, I'll provide a complete proof—the general method is important to remember.
The key idea is to apply the fundamental identity$$\sum_{d\mid m} \mu(d) = \begin{cases} 1, &\text{if } m=1, \\ 0, &\text{if } m>1\end{cases}$$with$m=\gcd(j,n)$. For any real numbers$x<y$, we obtain\begin{align*}\#\{ x < j \le y\colon \gcd(j,n)=1\} &= \sum_{x<j\le y} \sum_{d\mid\gcd(j,n)} \mu(d) \\&= \sum_{d\mid n} \mu(d) \sum_{\substack{x<j\le y \\ d\mid j}} 1 \\&= \sum_{d\mid n} \mu(d) \biggl( \Bigl\lfloor \frac yd\Bigr\rfloor - \Bigl\lfloor \frac xd\Bigr\rfloor \biggr) \\&= \sum_{d\mid n} \mu(d) \biggl( \frac{y-x}d + O(1) \biggr) \\&= (y-x) \sum_{d\mid n} \frac{\mu(d)}d + O\biggl( \sum_{d\mid n} |\mu(d)| \biggr) \\&= (y-x)\frac{\phi(n)}n + O(2^{\omega(n)}),\end{align*}where$\omega(n)$ is the number of distinct prime factors of$n$.
In particular,$\phi(n) \gg_\delta n^{1-\delta}$ and$2^{\omega(n)} \ll_\delta n^\delta$ for every$\delta>0$, so the above provides an asymptotic formula as soon as the interval$(x,y]$ has length greater than$n^\delta$. In particular, taking$(x,y] = (0,\varepsilon n]$ handles the question in the OP.
- 1$\begingroup$There's a typo in the last equality, it should be $\phi(n)/n$, right?$\endgroup$Bruno Andrades– Bruno Andrades2025-11-20 20:48:58 +00:00CommentedNov 20 at 20:48
- $\begingroup$Right, @BrunoAndrades, $\sum_{d \mid n} \frac{\mu(d)}{d} = \frac{\varphi(n)}{n}$.$\endgroup$Dermot Craddock– Dermot Craddock2025-11-20 21:13:49 +00:00CommentedNov 20 at 21:13
- $\begingroup$Thanks for the answer! That's a nice trick to know$\endgroup$Bruno Andrades– Bruno Andrades2025-11-20 22:15:16 +00:00CommentedNov 20 at 22:15
You mustlog in to answer this question.
Explore related questions
See similar questions with these tags.